QDV 15 & QDV 16: 2013, treize, treize

Bonjour à toutes et à tous;

En cette nouvelle année $2013$, le Comité du Vendredi vous présente ses meilleurs voeux de bonheur, de santé et d'épanouissement mathématique.

Tradition oblige, cette première QDV de l'année $2013$ tournera donc autour du nombre $2013$.

2013 (1) :
Quelle est la décomposition en facteurs premiers de $2013~ ?~~ 2014~ ?~~ 2015~ ?$
En quelle année retrouvera-t-on cette particularité ?
Si votre réponse est trouvée sans ordinateur, merci de proposer votre solution, si elle est trouvée avec MAPLE, merci de proposer votre procédure.
Même question pour quatre années successives ?

2013 (2) :
Soit $g$ une fonction de $\N$ dans $\N$ telle que $g(g(n))=3n$ et $g(n+1) > g(n)$
Calculer $g(2013)$.

Amicalement. Bernard p/o Le Comité Du Vendredi.

PS: Le Comité de Vendredi vous présente également ses excuses pour vendredi dernier: la question a été oubliée!

Réponses

  • 2013=3*(667+4)=3*671=3*11*61.
    2014=2*1007=2*19*53.
    2015=5*403=5*13*31.

    J'imagine que la propriété remarquable est que ces trois nombres sont des 2-libres produits de 3 facteurs premiers consécutifs.
    Pour la prochaine année, je table sur 2013+30k pour un certain k.
  • 2013 (2) : c'est 3852. En effet, on a $g(0)=0$, $g(1)=2$, $g(2)=3$, et on montre par récurrence que
    $g(3^k-j)=2(3^k)-3j$ pour $j=0,...,3^{k-1}$,
    $g(3^k+j)=2(3^k)+j$ pour $j=0,...,3^k$.

    En particulier, $g(3^7-174)=2\times 3^6-3\times 174=3852$.
  • Bonjour à tous,

    2013 (1) : Si la particularité attendue est simplement d'être le produit de trois nombres premiers, alors la prochaine année est $2022=2\times 3\times 337$.
    readlib(ifactors);
    for n from 2016 do
    if add(x[2],x=ifactors(n)[2])=3 then print(n):break fi;
    od:
    

    En fait nous sommes actuellement dans un cycle de quatre années consécutives de ce type puisque $2012=2\times2\times 503$.

    Les prochaines sont $2091$, $2092$, $2093$ et $2094$ : $2091=3\times 17\times 41$, $2092=2\times 2\times 523$, $2093=7\times 13\times 23$ et $2094=2\times 3\times 349$. Les précédentes étaient $602$, $603$, $604$, $605$ et $1083$, $1084$, $1085$, $1086$.

    Évidemment, il n'est pas possible de trouver quatre années consécutives produits de trois facteurs premiers distincts, puisque l'une de ces quatre années serait divisible par $4$.
  • Bonsoir,

    2013 (1) : la propriété attendue est que $2013$ est le premier de trois entiers consécutifs qui sont le produit de trois nombres premiers différents, voir le message de Sylvain, ce qui n'est effectivement pas possible pour quatre entiers consécutifs comme l'a rappelé Juge Ti. En quelle année retrouvera-t-on donc trois entiers consécutifs qui soient le produit de trois nombres premiers différents ?

    2013 (2) : JLT, es-tu certain du $3852$ ?

    [Moralité : ne jamais intervenir vite fait sur le forum entre deux plats quand il y a réception à la maison (td) ]

    2013 (3) Soit la suite $(u_n)_n$ définie par $u_1=1$et pour $n \geq1~~:~~ u_{n+1}=\dfrac{u_n}{1+nu_n}$
    Calculer $u_{2013}$.

    2013 (4)
    Existe-t-il $\ell$ et $m$ tels que : $\displaystyle \dfrac {1}{2013} = \sum_{k=\ell}^{\ell+m} \dfrac{1}{k(k+1)}$ ?
    Si oui, $\ell= ?$ et $m=?$

    Bonne nuit, amicalement.
  • 2013 (1) : Ah d'accord. Alors ce sera en $2665$ : on a $2665=5\times13\times41$, $2666=2\times31\times43$ et $2667=3\times7\times127$. Voilà la procédure :
    readlib(ifactors);
    c:=0:
    for n from 2016 do
    l:=ifactors(n)[2]:
    if add(x[2],x=l)=3 and nops(l)=3 then c:=c+1:if c=3 then print(n-2,n-1,n):break fi: else c:=0 fi;
    od:
    
  • 2013 (3) : Je pose $v_n=1/u_n$. Alors $v_1=1$ et, pour tout $n\in\N$, $v_{n+1}=v_n+n$.

    Par conséquent, $v_n=1+1+2+\ldots+(n-1)=1+\dfrac{n(n-1)}{2}$ et donc $u_n=\dfrac{2}{2+n(n-1)}$.

    En particulier $u_{2013}=\dfrac{1}{2025079}$.
  • 2013 (4) : On a $\displaystyle\sum_{k=l}^{l+m} \dfrac{1}{k(k+1)}=\sum_{k=l}^{l+m} \left(\dfrac{1}{k}-\dfrac{1}{k+1}\right)=\dfrac{1}{l}-\dfrac{1}{l+m+1}=\dfrac{m+1}{l(l+m+1)}$.

    L'équation posée est donc équivalente à l'équation diophantienne $2013(m+1)=l(l+m+1)$.

    Là je triche et je demande à Maple qui me donne généreusement les couples solutions : $(924, 783)$, $(1342, 2683)$, $(1464, 3903)$, $(1650, 7499)$, $(1830, 18299)$, $(1892, 29583)$, $(1914, 37003)$, $(1952, 62463)$, $(1980, 118799)$, $(2002, 364363)$, $(2004, 446223)$, $(2010, 1346699)$ et $(2012, 4048143)$.

    Si j'ai le courage j'essaierai de le faire à la main (mais, franchement, ç'aurait été plus facile en 2011 !).
  • 2013 (4) A la main, l'équation diophantienne $2013(m+1)=l(l+m+1)$ se traite sans mal. En posant $a= 2013$ et $p=m+1$, cette équation devient : $ap=l(l+p)$, soit : $a^{2}-l^{2}+ap-lp=a^{2}$, ou enfin : $(a-l)(a+l+p)=a^{2}$. Comme $a-l<a+p+l$, le nombre de solutions est la petite moitié du nombre de diviseurs de $a^{2}=2013^{2}=7^{2}\times 11^{2}\times 61^{2}$, soit 13, c'est ce que dit tonton Maple.
    Bonne journée.
    RC
  • 2013 (2) Ceci se trouve ici :
    http://2000clicks.com/mathhelp/PuzzleIncreasingIntegerFunctionAnswer.aspx
    et là :
    http://www.fmat.cl/index.php?showtopic=11703
    J'aui déjà dit que je ne voyais pas l'intérêt de poser ici des problèmes directement recopiés de problèmes déjà posés lors d'olympiades ou de leur preparation, et dont la solution est disponible publiquement, notamment sur Internet, mais enfin, les goûts et les couleurs, ça ne se discute pas.
    Le résultat 3852 me semble correct.
    Bonne dimanchade.
    RC
  • Bonjour à tou(te)s,

    (tu) 2013 (1) Juge Ti 2013 (3) (tu)

    (tu) Juge Ti 2013 (4) RC (tu)

    Le 2013 (3) provient d'un problème des Olympiades islandaises de 1996.

    Le 2013 (4) trouve son inspiration dans l'un des problèmes du Canadian Mathematical Olympiad de 1973.

    2013 (5) Si le nombre $x$ est tel que $x + \dfrac{1}{x}=-1$, que vaut $x^{2013}+\dfrac{1}{x^{2013}} =~~ ?$

    2013 (6) : lors du 2013 (1), on s'est aperçu que $2013, 2014$ et $2015$ sont tous trois le produit de trois nombres premiers distincts et que cela se reproduira, même si peut-être nous ne serons plus sur cette terre, en $2665, 2666, 2667$.
    On a donc $\tau(2013)=\tau(2014)=\tau(2015)=8$. Quand retrouvera-t-on trois entiers consécutifs possédant le même nombre de diviseurs ? ...et pour quatre entiers consécutifs ?
    A la main, ou avec l'érable, et alors avec une procédure.

    2013 (7) Trouver $N \in \N^{*}$ vérifiant $\big( \sqrt{2013} - \sqrt{2012} \big)^{2014} = \sqrt{N} -\sqrt{N-1}$

    Amicalement.
  • bs a écrit:
    JLT, es-tu certain du $ 3852$ ?

    Je crois, oui. J'ai vérifié avec Scilab.
    -->g=zeros(1:10000);
    
    -->for k=1:9;for j=0:3^k;n=3^k+j;if n<=10000 then g(n)=2*3^k+j;end;end;end
    
    -->for k=1:9;for j=0:3^(k-1);n=3^k-j;if n <=10000 then g(n)=2*3^k-3*j;end;end;end
    
    -->g(2013)
     ans  =
     
        3852.  
    

    Je vérifie que $g(g(n))=3n$ pour $n\le 5000$ :
    -->n=1:5000;max(abs(g(g(n))-3*n))
     ans  =
     
        0.  
    

    Je vérifie que $g$ est strictement croissante :
    -->min(g(2:10000)-g(1:9999))
     ans  =
     
        1.  
    

    Les premières valeurs de $g(n)$ pour $1\le n\le 100$ sont
           column  1 to 14
     
        2.    3.    6.    7.    8.    9.    12.    15.    18.    19.    20.    21.    22.    23.  
     
             column 15 to 27
     
        24.    25.    26.    27.    30.    33.    36.    39.    42.    45.    48.    51.    54.  
     
             column 28 to 40
     
        55.    56.    57.    58.    59.    60.    61.    62.    63.    64.    65.    66.    67.  
     
             column 41 to 53
     
        68.    69.    70.    71.    72.    73.    74.    75.    76.    77.    78.    79.    80.  
     
             column 54 to 66
     
        81.    84.    87.    90.    93.    96.    99.    102.    105.    108.    111.    114.    117.  
     
             column 67 to 78
     
        120.    123.    126.    129.    132.    135.    138.    141.    144.    147.    150.    153.  
     
             column 79 to 90
     
        156.    159.    162.    163.    164.    165.    166.    167.    168.    169.    170.    171.  
     
             column  91 to 100
     
        172.    173.    174.    175.    176.    177.    178.    179.    180.    181.  
    
  • 2013 (5)

    Si le nombre $ x$ est tel que $ x + \dfrac{1}{x}=-1$ alors $x=j$ ou $j^2$ donc $ x^{2013}+\dfrac{1}{x^{2013}} =2$.
  • Re,
    J'aui déjà dit que je ne voyais pas l'intérêt de poser ici des problèmes directement recopiés de problèmes déjà posés lors d'olympiades ou de leur preparation,

    Ben... peut-être que je ne possède pas assez de créativité pour inventer de tels nouveaux exercices ;)

    Raymond, maintenant, si tu souhaites créer un exercice avec 2013 et le proposer dans ce fil, c'est bien volontiers que nous essaierons de le résoudre.

    Chaque fin d'année, pour préparer la première QDMachin de la nouvelle année à venir, il faut essayer de nombreux exercices déjà existants, et adapter l'énoncé, parfois même après adaptation, il n'est pas possible de créer un nouvel énoncé et donc il faut attendre une autre année plus favorable, mais c'est très plaisant.

    Merci pour votre participation.

    (tu) 2013 (2) JLT 2013 (5) (tu)

    Amicalement.
  • 2013 (7) : soit $n=2013$. Alors c'est
    $$\sqrt{N}=\dfrac{(\sqrt{n}+\sqrt{n-1})^{n+1}+(\sqrt{n}-\sqrt{n-1})^{n+1}}{2}.$$
  • 2013 (6) : Les trois prochains entiers consécutifs possédant le même nombre de diviseurs sont $2054=2\times13\times79$, $2055=3\times5\times137$ et $2056=2^3\times257$. On en aura quatre consécutifs en $3655$, $3656$, $3657$ et $3658$.
    with(numtheory):
    c:=0:
    for n from 2015 do
    if tau(n)=tau(n+1) then c:=c+1:if c=2 then print(n):break fi; else c:=0 fi;od:
    
  • Pour 2013 (7) on peu utiliser les polynômes de Tchebychev.
    Plus généralement montrons que $(\sqrt n-\sqrt{n-1})^p=\sqrt N_p-\sqrt{N_p-1}$ avec $N_p\in\N$.
    Pour cela posons $n=\cosh^2t$.
    $(\sqrt n-\sqrt{n-1})^p=e^{-pt}=\cosh(pt)-\sinh(pt)$ d'où $N_p=\cosh^2(pt)=T_p^2(\cosh t)$ si $T_p$ est le polynôme de Tchebychev de première espèce (à coefficients entiers) vérifiant $\cosh(pt)=T_p(\cosh t)$.
    Comme $T_p$ a la parité de $p$, $T_p^2$ est un polynôme pair et on a donc $N_p=Q_p(n)$ avec $Q_p$ polynôme de degré $p$ à coefficients entiers.

    On peut obtenir une expression simple en distinguant les cas $p$ pair et $p$ impair et en utilisant $\cosh(2t)=2\cosh^2t-1=2n-1$.

    Si $p=2q$: $N_{2q}=\cosh^2(2qt)=T_q^2(\cosh(2t))$ donc $N_{2q}=T_q^2(2n-1)$.

    Si $p=2q+1$: $N_{2q+1}=\cosh^2((2q+1)t)=n\left(\dfrac{\cosh((2q+1)t)}{\cosh t}\right)^2$.
    On a $\dfrac{\cosh((2q+1)t)}{\cosh t}=\dfrac{\sinh((2q+2)t)-\sinh(2qt)}{\sinh(2t)}=U_{q+1}(\cosh(2t))-U_p(\cosh(2t))$ où $U_p$ est le polynôme de Tchebychev de seconde espèce (à coefficients entiers) vérifiant $\dfrac{\sinh(pt)}{\sinh t}=U_p(\cosh t)$.
    On obtient donc $N_{2q+1}=n(U_{q+1}(2n-1)-U_q(2n-1))^2$.

    Les premières valeurs sont: $N_2=(2n-1)^2$, $N_3=n(4n-3)^2$, $N_4=(8n^2-8n+1)^2$, $N_5=n(16n^2-16n+1)^2$.

    Pour l'application numérique $n=2013$ et $p=2014$ Maple affiche un entier de 7866 chiffres.
  • Bonjour,

    (tu) JLT (tu) jandri (tu) pour la 2013 (7) et sa généralisation !

    (tu) Juge Ti pour la 2013 (6)

    Quelques sources :

    2013 (1) : https://oeis.org/search?q=A066509&sort=&language=english&go=Search
    2013 (5) : Olympiades Chiliennes de 1994/1995
    2013 (6) : http://oeis.org/search?q=A005238&sort=&language=english&go=Search
    2013 (7) : Compétition Hongrie-Israel de 1997


    2013 (8) : A quoi est égal le produit des racines de l'équation ci-dessous ?

    $$\sqrt{2013} \times x^{\log_{2013}x} = x^6$$

    2013 (9) : Calcul mental : quel est le plus petit nombre $2n$ tel que $2013$ divise $2 \times 4 \times \dots \times 2n~~?$

    2013 (10) : Combien de termes différents contient la suite finie $\left\lfloor\dfrac{n^2}{2013}\right\rfloor $ pour $n=1,2,...,2013$ ?

    Amicalement.
  • 2013 (8) : soit $n=2013$ et $X=\log x$. En passant au log l'équation on a
    $$\frac{1}{2}\log n + \frac{X^2}{\log n}-6 X=0$$
    donc $X^2-6(\log n) X +\cdots =0$. Il y a deux solutions en $X$ dont la somme vaut $6\log n$, donc le produit des solutions en $x$ vaut $n^6$.
  • 2013 (9) : $2013=3\times 11\times 61$ donc $n=61$.
  • 2013 (10) : 1510.
    Plus généralement j'ai vérifié (pour $n\le2013$) que la suite finie $ \left\lfloor\dfrac{k^2}n\right\rfloor $ pour $1\le k\le n $ contient $ 1+\left\lfloor\dfrac{3n}{4}\right\rfloor $ termes distincts.
  • Bonjour,

    (tu) jandri pour la généralisation du 2013 (10) ...si l'un d'entre vous possède une preuve de ce résultat, merci de la proposer.

    Quelques provenances; depuis l'éclatement de la Yougoslavie, de la Tchécoslovaquie et de l'URSS il y a beaucoup plus de choix.

    2013 (8) : Moldavie en 1996.

    2013 (9) : Alberta.

    2013 (11) : un nombre est dit triangulé s'il est triangulaire ou somme de nombres triangulaires...tous différents (merci Juge Ti). Combien de nombres triangulés figurent dans l'ensemble $\{1,2,\dots, 2013\} ?$

    2013 (12) : n'oublions pas le fil Une partie entière et 2013 de notre ami Cidrolin.

    Amicalement.
  • 2013 (11) : Facile : il y en a $2013$, puisque tout entier naturel non nul peut s'écrire comme somme de $1s$, et $1$ est un nombre triangulaire ! ;)

    Maintenant si tu voulais dire somme de nombres triangulaires distincts, alors d'après http://oeis.org/A053614 il y en a $2007$.

    Je me permets de rajouter une question que je me suis posée hier et qui m'a donné du mal (je manque de pratique), mais qui ne devrait pas demander plus de trente secondes à nos algébristes chevronnés :

    2013 (13) : Combien y a-t-il de groupes finis d'ordre 2013 (à isomorphisme près bien entendu) ?
  • Salut les copains,

    Je profite de l'occasion pour recycler celle-ci, pour ceux qui ne l'ont pas vue passer

    2013 (33) : Comment écrirait-on 2013 en notation sumérienne?

    A l'attention de Raymond, je précise que 2013 n'est pas une année olympique, mais je souhaite à tous qu'elle soit féconde sur le plan mathématique entre autres.
  • 2013 (13) : il y a deux groupes d'ordre 2013 à isomorphisme près. En effet, soit $G$ un groupe d'ordre 2013. En utilisant les théorèmes de Sylow on voit qu'il y a un et un seul $11$-Sylow $S_{11}$ et un et un seul $61$-Sylow $S_{61}$.

    La conjugaison donne un morphisme $S_{11}\to \mathrm{Aut}(S_{61})\cong \Z/60\Z$ qui est nécessairement trivial puisque $11$ et $60$ sont premiers entre eux, donc les éléments de $S_{11}$ commutent avec ceux de $S_{61}$.

    Il y a donc un sous-groupe distingué $N$ isomorphe à $\Z/11\Z\times \Z/61\Z$. Soit $H$ un $3$-Sylow. Alors $G$ est un produit semi-direct de $H$ par $N$. Comme $\mbox{Aut}(N)\cong \Z/{10}\Z\times \Z/60\Z$ et que tout morphisme $H\to \Z/{10}\Z$ est trivial, on voit que $G$ est produit d'un groupe cyclique d'ordre 11 avec un groupe d'ordre $3\times 61$. Comme il y a un et un seul groupe non abélien d'ordre $3\times 61$, on obtient le résultat : $G$ est cyclique ou bien est isomorphe à $\Z/{11\Z}\times (\Z/61\Z\rtimes \Z/3\Z)$.
  • Pour la généralisation de jandri du 2013 (9), soit $k=[\frac{n+1}{2}]$. Si $j\le k$ alors $\frac{j^2}{n}-\frac{(j-1)^2}{n}\le 1$, donc $[\frac{j^2}{n}]-[\frac{(j-1)^2}{n}]\le 1$, ce qui montre que tous les entiers entre 0 et $[\frac{k^2}{n}]$ sont atteints.

    Ensuite, pour $j>k$, on a $\frac{j^2}{n}-\frac{(j-1)^2}{n}> 1$ donc tous les $[\frac{j^2}{n}]$ pour $j\ge k$ sont distincts.

    Cela donne au total $1+[\frac{k^2}{n}]+(n-k)$ valeurs.

    Pour $n=4m$ on a $k=2m$ et $1+[\frac{k^2}{n}]+(n-k)=3m+1$.

    Pour $n=4m+1$ on a $k=2m+1$ et $1+[\frac{k^2}{n}]+(n-k)=3m+1$.

    Pour $n=4m+2$ on a $k=2m+1$ et $1+[\frac{k^2}{n}]+(n-k)=3m+2$.

    Pour $n=4m+3$ on a $k=2m+2$ et $1+[\frac{k^2}{n}]+(n-k)=3m+3$.

    Dans tous les cas on retrouve la formule de jandri.
  • Merci JLT pour le 2013 (13) qui était plus subtil que je ne pensais.

    2013 (33) : On a $2013=33\times 60+33$, donc en sumérien ça donne :

    \begin{pspicture}(0,-0.53)(5.0,0.53)
    \psline[linewidth=0.04cm,arrowsize=0.05291667cm 3.0,arrowlength=1.25,arrowinset=0.2]{-<}(4.38,-0.49)(4.38,0.51)
    \psline[linewidth=0.04cm,arrowsize=0.05291667cm 3.0,arrowlength=1.25,arrowinset=0.2]{-<}(4.68,-0.49)(4.68,0.51)
    \psline[linewidth=0.04cm,arrowsize=0.05291667cm 3.0,arrowlength=1.25,arrowinset=0.2]{-<}(4.98,-0.49)(4.98,0.51)
    \psline[linewidth=0.04cm](3.12,-0.01)(4.12,0.49)
    \psline[linewidth=0.04cm](3.12,-0.01)(4.12,-0.51)
    \rput(-0.02044586, 0.0){\psdots[dotsize=0.12,dotangle=-90.0,dotstyle=diamond*](3.24,0)}
    \psline[linewidth=0.04cm](2.87,-0.01)(3.87,0.49)
    \psline[linewidth=0.04cm](2.87,-0.01)(3.87,-0.51)
    \rput(-0.02044586, 0.0){\psdots[dotsize=0.12,dotangle=-90.0,dotstyle=diamond*](2.99,0)}
    \psline[linewidth=0.04cm](2.62,0.01)(3.62,0.49)
    \psline[linewidth=0.04cm](2.62,0.01)(3.62,-0.51)
    \rput(-0.02044586, 0.0){\psdots[dotsize=0.12,dotangle=-90.0,dotstyle=diamond*](2.74,0)}
    \psline[linewidth=0.04cm,arrowsize=0.05291667cm 3.0,arrowlength=1.25,arrowinset=0.2]{-<}(1.76,-0.49)(1.76,0.51)
    \psline[linewidth=0.04cm,arrowsize=0.05291667cm 3.0,arrowlength=1.25,arrowinset=0.2]{-<}(2.06,-0.49)(2.06,0.51)
    \psline[linewidth=0.04cm,arrowsize=0.05291667cm 3.0,arrowlength=1.25,arrowinset=0.2]{-<}(2.36,-0.49)(2.36,0.51)
    \psline[linewidth=0.04cm](0.5,-0.01)(1.5,0.49)
    \psline[linewidth=0.04cm](0.5,-0.01)(1.5,-0.51)
    \rput(-0.02044586, 0.0){\psdots[dotsize=0.12,dotangle=-90.0,dotstyle=diamond*](0.62,0)}
    \psline[linewidth=0.04cm](0.25,-0.01)(1.25,0.49)
    \psline[linewidth=0.04cm](0.25,-0.01)(1.25,-0.51)
    \rput(-0.02044586, 0.0){\psdots[dotsize=0.12,dotangle=-90.0,dotstyle=diamond*](0.37,0)}
    \psline[linewidth=0.04cm](0.0,0.01)(1.0,0.49)
    \psline[linewidth=0.04cm](0.0,0.01)(1.0,-0.51)
    \rput(-0.02044586, 0.0){\psdots[dotsize=0.12,dotangle=-90.0,dotstyle=diamond*](0.12,0)}
    \end{pspicture}
  • (tu) Juge Ti pour le 33'33"
  • .... nous vous proposons de prolonger la QDV 15 en QDV 16 pour continuer à fêter dignement 2013.

    2007 (14) Parmi toutes les formes quadratiques s'écrivant $x^2+px+q$ avec $ 1 \leq p \leq 2013$ et $ 1 \leq q \leq 2013$, considérons les deux ensembles particuliers suivants :
    $A)$ : les trinômes possédant des racines entières,
    $B)$ : les trinômes ne possédant pas de racines réelles.
    Quel ensemble a le plus grand cardinal ?

    2007 (15) Une fonction $f$ vérifie pour tout entier $n$ positif : $$f(1)+f(2)+\dots+f(n)=n^2f(n).$$ Si $f(1) = 1007$, que vaut $f(2013)~~ ?$

    Amicalement. Bernard p/o Le Comité Du Vendredi.
  • 2013 (15) : $f(n)=\dfrac{2014}{n(n+1)}$, donc $f(2013) =\dfrac{1}{2013}$.
  • 2013 (14) A la louche je dirais B).

    En effet, notons $n=2013$. Il n'y a pas de solution réelle si $1\le p\le n$ et $\dfrac{p^2}{4}< q\le n$. Pour $p\le \sqrt{n}$, tous les $q$ compris entre $\dfrac{n}{4}$ et $n$ marchent, donc il y a au moins $\dfrac{3}{4}n^{3/2}$ possibilités.

    Par contre, il y a des solutions entières lorsque $q=rm$ et $p=r+m$ pour certains entiers positifs $r$ et $m$ tels que $1\le r+m\le n$ et $1\le rm\le n$. Le nombre de possibilités est inférieur au nombre de couples $(r,m)$ d'entiers positifs vérifiant $rm\le n$ qui vaut environ $n\log n$.
  • Pour compléter le {\bf 2013 (13)} de Juge TI et la réponse de JLT, on sait que le nombre $g(n)$ de (classes d'isomorphismes) de groupes d'ordre $n$ vérifie

    $$g(n) \leqslant n^{\{2/27+o(1)\}m^2}$$

    où $m = \underset{p \mid n}{\max} v_p(n)$, avec égalité si $n$ est une puissance d'un nombre premier (et le terme d'erreur est alors affiné). Ici $m=1$ et $2013^{2/27} \approx 1,757$.

    De même, si $a(n)$ désigne le nombre de (classes d'isomorphismes) de groupes {\it abéliens} d'ordre $n$, alors il est assez facile de montrer, en utilisant les théorèmes de structure de ces groupes, que, si $n=p_1^{e_1} \dotsb p_r^{e_r}$, alors

    $$a(n) = \prod_{k=1}^r P(e_k)$$

    où $P(m)$ désigne le nombre de partitions de l'entier $m \geqslant 1$. On en déduit que $a(2013) = 1$, en concordance avec le résultat obtenu par JLT.
  • Bonsoir,

    (tu) JLT pour la preuve de la généralisation proposée par jandri au 2013 (10) question venue tout droit des Balkans en 2001.

    (tu) JLT pour la 2013 (15), en fait il fallait calculer $f(2013)$, lapsus dans l'énoncé, exercice proposé en Estonie en 1996.

    (tu) pour la louche de JLT au 2013 (4), et sans louche ?

    Merci enonce pour les compléments que tu apportes au 2013 (13).

    2013 (16) : soit $a_1, a_2, \dots, a_n$, $n \geq 2$, des entiers réels positifs vérifiant $$\dfrac{1}{2013+a_1}+\dfrac{1}{2013+a_2}+\dots+\dfrac{1}{2013+a_n}= \dfrac{1}{2013}.$$ Montrer que $\dfrac{\sqrt[n]{a_1a_2...a_n}}{n-1} \geq 2013.$

    2013 (17) : soit $m$ et $n$ le nombre de chiffres utilisés dans l'écriture respective de $2^{2013}$ et $5^{2013}$ en base $10$.
    Que vaut $m+n=$:S

    2013 (18) : on considère le produit $P$ de tous les multiples de $6$ inférieurs à $2013$. Par combien de zéros se termine l'écriture du nombre $P$ ?

    Amicalement.
  • 2013 (17) : Le nombre de chiffres en base $10$ d'un entier naturel $x$ est $E\left(\dfrac{\ln x}{\ln 10}\right)+1$ puisque $10^{p-1}\leq x<10^{p}$ équivaut à $p-1\leq\dfrac{\ln x}{\ln 10}<p$.

    Par conséquent $2^{2013}$ a $m=E\left(\dfrac{2013\ln 2}{\ln 10}\right)+1=606$ chiffres et $5^{2013}$ en a $n=E\left(\dfrac{2013\ln 5}{\ln 10}\right)+1=1408$, et donc $m+n=2014$.

    On peut aussi être malin et écrire que $10^{m-1}<2^{2013}<10^{m}$ et que $10^{n-1}<5^{2013}<10^{n}$, donc $10^{m+n-2}<10^{2013}<10^{m+n}$, d'où $m+n=2014$.
  • Le {\bf 2013 (17)} ne vaut-il pas

    $$2 + \left \lfloor \log_{10} \left ( 2^{2013} \right ) \right \rfloor + \left \lfloor \log_{10} \left( 5^{2013} \right) \right \rfloor = 2014 \, ?$$
  • {\bf Théorème} Enonce tape moins vite que Juge TI...
  • Bon, moi aussi je vexu jouer,; mais faut être rapide ici...

    Pour le {\bf 2013 (18)}, on a $P=6^{335} \times 335 !$ et donc le nombre cherché vaut via la formule de Legendre

    $$v_5(335!) = \sum_{k=1}^{\infty} \left \lfloor \frac{335}{5^k} \right \rfloor = 82.$$

    C'est ça ?
  • @ enonce : :)-D

    2013 (18) : Ce sont les multiples de $30$ qui vont donner des zéros à la fin, mais ils n'en donnent pas tous autant.

    $750$ et $1500$ en donnent trois chacun puisque $750\times4=1500\times2=3000$. Ça nous fait déjà six zéros.

    Les autres multiples de $150$ en donnent deux chacun puisque $150\times 2=300$. Il y en a $11$, donc ça nous fait $22$ zéros de plus.

    Les autres multiples de $30$ ne donnent qu'un zéro chacun, il y en a $67-11-2=54$.

    Total : $54+22+6=82$ zéros.
  • Ah, là, je t'ai battu, Juge...
  • Argh... Ça fait un partout ! Je suis curieux de savoir combien de secondes d'écart il y a entre nos deux messages !
  • Aucune idée, mais j'ai tapé tellement vite que ma première phrase est bourrée de coquilles...Mais faut dire que tu ne m'as pas laissé le choix ! :)-D
  • 2013 (19)

    Pour $k \geq 1$ on pose $I_k=[ 33^k;33^{k+1}]$ et $J_k=[ 61^k;61^{k+1}]$.

    Montrer que l'intervalle $I_k$ contient une puissance de $2013$ si et seulement

    si l'intervalle $J_k$ n'en contient pas.
  • Bonjour Édouard,

    2013 (19) : Soit $k$ un entier naturel non nul.

    On a : $2013^m\in I_k\Leftrightarrow k\leq m\dfrac{\ln 2013}{\ln 33}\leq k+1\Leftrightarrow m-\alpha\leq\alpha k\leq m$ où $\alpha=\dfrac{\ln 33}{\ln 2013}$. Ainsi, si $m$ existe, on a nécessairement $m=E(\alpha k)+1$.

    De même : $2013^n\in J_k\Leftrightarrow k\leq n\dfrac{\ln 2013}{\ln 61}\leq k+1\Leftrightarrow n-\beta\leq\beta k\leq n$ où $\beta=\dfrac{\ln 61}{\ln 2013}$. Ainsi, si $n$ existe, on a nécessairement $n=E(\beta k)+1$.

    On a $\alpha+\beta=1$ car $2013=33\times 61$. De plus, puisque $\alpha$ et $\beta$ sont irrationnels, je peux remplacer toutes les inégalités larges par des inégalités strictes.

    Posons donc $m=E(\alpha k)+1$ et $n=E(\beta k)+1$.

    Alors $m-1<\alpha k<m$, donc on a soit $(1):m-1<\alpha k<m-\alpha$, soit $(2):m-\alpha<\alpha k<m$.

    Et de même $n-1<\beta k<n$, donc on a soit $(1'):n-1<\beta k<n-\beta$, soit $(2'):n-\beta<\beta k<n$.

    Mais en additionnant $(1)$ et $(1')$ on obtient $m+n-2<k<m+n-1$ : impossible. De même, en additionnant $(2)$ et $(2')$ on obtient $m+n-1<k<m+n$ : impossible.

    Ainsi on a soit $(1')$ et $(2)$, auquel cas $I_k$ contient une puissance de $2013$ mais $J_k$ n'en contient pas, soit $(1)$ et $(2')$, auquel cas $J_k$ contient une puissance de $2013$ mais $I_k$ n'en contient pas.
  • Bravo Juge Ti (tu)

    Cordialement

    Edouard Cidrolin
  • Intéressant et pas évident, le {\bf 2013 (16)} (en tout cas pour moi qui ne suis pas très fort).

    Pour ne pas avoir cherché pour rien, je voudrais soumettre les idées que j'ai eues mais qui n'aboutissent pas.

    Soient $m \leqslant H \leqslant G \leqslant A \leqslant M$ respectivement le minimum, les moyennes harmoniques, géométriques, arithmétiques et le maximum des réels positifs $a_i$, et je mets un "prime" pour les mêmes quantités concernant les réels $a_i + 2013$.

    On veut donc montrer que $G \geqslant 2013 (n-1)$.

    Ce qui suit est facile à obtenir, je ne détaille donc pas.

    $A' = A + 2013$, $H' = 2013 n$, $G e^{1/n} \leqslant G ' \leqslant G e^{2013/H}$ et $m \leqslant 2013(n-1) \leqslant M$.

    L'inégalité $a_i+2013 \geqslant a_i$ implique facilement $H \leqslant 2013n = H'$ (l'inégalité $H \leqslant H'$ aurait aussi pu se voir directement). Donc, si $H \geqslant 2013(n-1)$, le problème est résolu via $G \geqslant H$.

    On suppose donc que $H < 2013(n-1)$. J'ai obtenu les inégalités suivantes :

    (i) L'inégalité $A' \geqslant H'$ donne immédiatement $A \geqslant 2013(n-1)$.

    (ii) L'inégalité $H' \leqslant G'$ donne $G \geqslant 2013 n e^{-2013/H}$, qui, malheureusement, est moins bonne que $G \geqslant 2013 (n-1)$.

    J'en suis là ! D'autres idées ?
  • Bonjour,

    (tu) Joli tir groupé du Juge Ti et d'enonce pour les 2013 (17) et 2013 (18).
    Le 2013 (18) provient du Chili (1994).

    (tu) Edouard pour ses créations mathématiques originales autour de 2013.

    2013 (20) Pour chaque jour de l'année 2013 on calcule l'expression $x = j^m-a$ où $j, m$ et $a$ représentent respectivement le jour, le mois et l'année. On recherche les jours de 2013 pour lesquels $v_3(x)$ est le plus grand ?

    Bon dimanche, amicalement.
  • 2013 (16) : quitte à faire le changement de variables $a_i=2013 x_i$, on se ramène à montrer que si $\sum_{i=1}^n\dfrac{1}{1+x_i}=1$ alors $x_1\cdots x_n\ge (n-1)^n$, ou encore que $\sum_{i=1}^n \log x_i=n\log (n-1)$. Posons $y_i=\dfrac{1}{1+x_i}$. On a $0<y_i<1$ et $\sum_i y_i=1$, et il faut montrer que $\sum_{i=1}^n f(y_i)\ge n f(1/n)$ où $f(t)=\log((1-t)/t)$.

    On se place en un point $(y_1,\ldots,y_n)$ qui minimise $\sum_{i=1}^n f(y_i)$ sous la contrainte $y_i>0$, $\sum y_i=1$ (il n'est pas difficile de montrer qu'un tel point existe).

    Supposons par l'absurde qu'il existe $i$ et $j$ tels que $y_i<y_j$. Comme $y_i+y_j<1$, on a déjà $y_i<1/2$, et $y_i<y_j<1-y_i$. Comme $f'(t)=-\dfrac{1}{t(1-t)}$, on en déduit facilement que $f'(y_i)<f'(y_j)$, donc $\dfrac{d}{dt}(f(y_i+t)+f(y_j-t))<0$, par conséquent la somme $f(y_1)+\cdots+f(y_n)$ n'est pas minimale (sinon on pourrait remplacer $y_i$ par $y_i+t$ et $y_j$ par $y_j-t$ pour $t>0$ assez petit).

    On a donc $y_1=\cdots =y_n$, et comme $\sum y_i=1$, on en déduit que $y_i=\dfrac{1}{n}$ pour tout $n$ et donc que $\sum_{i=1}^n f(y_i)= n f(1/n)$.
  • Bien joué, JLT.
  • 2013 (20) : $2013=3\times 671$ est divisible par $3$, donc si $j$ n'est pas divisible par $3$, alors $x$ non plus.

    Soit $j=3^nk$ avec $n\geq 1$. Alors $x=3^{mn}k^m-2013=3(3^{mn-1}k^m-671)$. Mais $671$ n'est pas divisible par $3$, donc si $m>1$ ou $n>1$ alors $v_3(x)=1$.

    Pour $m=1$ et $n=1$ on a $x=3(k-671)$. Or $671$ est congru à $5$ modulo $9$, donc c'est le $15$ janvier $2013$ que $v_3(x)$ est le plus grand.
  • Bonsoir,

    (tu) pour le 2013 (16) proposé lors des Olympiades du Vietnam en 1998.

    (tu) pour le 2013 (20) proposé lors des Olympiades tchèques et slovaques de 1998.

    Merci pour toutes vos (bonnes) réponses.

    Amicalement.
Connectez-vous ou Inscrivez-vous pour répondre.